Last visit was: 25 Apr 2024, 02:23 It is currently 25 Apr 2024, 02:23

Close
GMAT Club Daily Prep
Thank you for using the timer - this advanced tool can estimate your performance and suggest more practice questions. We have subscribed you to Daily Prep Questions via email.

Customized
for You

we will pick new questions that match your level based on your Timer History

Track
Your Progress

every week, we’ll send you an estimated GMAT score based on your performance

Practice
Pays

we will pick new questions that match your level based on your Timer History
Not interested in getting valuable practice questions and articles delivered to your email? No problem, unsubscribe here.
Close
Request Expert Reply
Confirm Cancel
Magoosh GMAT Instructor
Joined: 28 Dec 2011
Posts: 4452
Own Kudos [?]: 28571 [0]
Given Kudos: 130
Magoosh GMAT Instructor
Joined: 28 Dec 2011
Posts: 4452
Own Kudos [?]: 28571 [0]
Given Kudos: 130
Magoosh GMAT Instructor
Joined: 28 Dec 2011
Posts: 4452
Own Kudos [?]: 28571 [0]
Given Kudos: 130
Magoosh GMAT Instructor
Joined: 28 Dec 2011
Posts: 4452
Own Kudos [?]: 28571 [0]
Given Kudos: 130
How to Get a Perfect GMAT Score [#permalink]
Expert Reply
FROM Magoosh Blog: How to Get a Perfect GMAT Score
The sub-4 minute mile; a perfect round of golf; a 300 in bowling…oh the elusive Mt. Everest. A GMAT 800: the perfect GMAT score.

Improbables of perfection, how can any ever hope to summit your slippery slopes?

Hard work won’t quite cut it. An element of luck, a touch of madness, and more than an average serving of raw talent might be enough.

If you’re aiming to get a GMAT perfect score, check out this SlideShare below, and read the following tips.



Perfect GMAT Score
In terms of attainability, where then does the GMAT perfect score fall – that perfect 800 we wish we could have emblazoned on our transcript? Well, fewer than 30 each year attain this coveted score. And as I am sure a few of these are tutors, that means even fewer test takers are scoring a perfect 800 on the GMAT.

To put things into perspective, 150 people climb Everest each year (though 5 do die – which is something I’ve never heard befall a GMAT test taker).

Clearly, a perfect score is extremely difficult – though not necessarily impossible. Below are some points to keep in mind if you have your eye on GMAT perfection.

GMAT Perfect Score: Some Criteria to Consider
Educational Background
The philosopher-physicist has it best. Strong quant skills coupled with formidable problem skills, this person is able to quickly see the big picture and discern the answer, that one tree in the dense forest of verbiage and equivocation.

Physicists are also adept at seeing some of the underlying patters in the verbal section. But it is the philosopher who truly excels. The critical reasoning section? Philosophers have been doing this kind of analytical thinking for years – and the level of critical reasoning philosophers have been engaging in makes the GMAT CR seem like a high school equivalency exam.

While the philosopher-physicist hybrid is the ideal, there are other fields of study that also handle the GMAT well. If you have a computer science background, you will have the problem solving skills necessary to do very well on the test. In general math types will fare well; even the verbal section is based on logic (think Sentence Correction). Yet, math types out there might want to make sure they are avid readers. If they have a penchant for literature, growing up with the likes of Dickens and Steinbeck, they will likely do well on the verbal. Ultimately, the student who will do best is one who blends analytical and mathematical thinking with a healthy dollop of verbal panache — whether they be an English major who does Sudoku in their spare time or an engineer with a thing for Jane Austen.

Avid Reading
Those who read challenging books and articles should have little problem navigating the dense, quasi-academic language of the GMAT. Proper idiomatic construction should flow from their tongues with the same insouciant ease most of us have discussing the weather. Grammatical niceties will jump out from the page, amidst the noise of GMAT distractors. (I say this based on my literary-inclined friends who find that they can rely on their ear to ace the Sentence Correction section — something even most native English speakers can’t do).

My advice: read challenging – but not utterly boring – works. And pay attention to language and turns of phrases. Additionally, it will help if you read widely. An article on species extinction in The New Yorker for breakfast and post-lunch reading of the latest Booker Prize-winning novel.

Meticulousness
The perfect scorer isn’t necessarily the fastest. But they are the most careful. The perfect scorer double-checks their answers and does not leap at an answer choice because it sounds good. He/she is aware of the traps that the GMAT lays, and is assiduously careful to avoid falling for them. Indeed, even perfect scorers aren’t always perfect scorers. Perhaps on their mock exams they missed a few questions.

Likely, once they saw their answer was incorrect, they could go back to the question and identify what was wrong with it. There mistake wasn’t conceptual; it was careless.

When you get close to perfection on the GMAT, it is likely meticulousness keeping you from an 800. Double-check that answer on Data Sufficiency (that notoriously and deceptively tricky section), don’t gloss over an answer choice on Reading Comprehension and end up missing that one critical word that invalidates the answer choice.

Meticulousness carries over to other facets of the GMAT besides answering questions. How you prepare is key to improving your score, especially when that score is the difference between 750 to 800 (those extra 50 points will require an extra serving of self-discipline.)

The perfect 800 scorer will be highly regimented and devise a GMAT study schedule. They will focus on their weaker areas and be as precise as possible when determining exactly why they missed those precious few points keeping them from scoring an 800. “I’m bad at hard science passages” is not the kind of mindset you need. Rather, “I will make sure to pay attention to how the author slips in a technical term mentioned earlier in the passage, instead of trying to justify an answer that I know is not quite right because I’m looking at the wrong part of the passage”. Essentially, perfect scorers are sticklers who look out for everything, from small math errors to grammatical nuances.

The Best Test Prep
To be able to attain a perfect score on the GMAT, you have to understand the subtleties of the test, and only official practice material will truly give you that. Yes, there is other material out there, some of which does a good job — Magoosh GMAT (yes, I know, conflict of interest), Manhattan GMAT (practice tests are even harder than the GMAT), and Veritas (does an okay job) — and there are the rest, from the benignly bad to the downright abominable, which will actually hurt your score.

This may be the first time you are hearing about Magoosh, and while we definitely can’t promise you a perfect score, we can promise you the best online video prep. You will nail the fundamentals (even if you weren’t a philosopher poet) and will have hundreds of practice questions to help you scale the Mt. Everest that is the GMAT.

How to Get a Perfect Score on the GMAT [/b] from Magoosh

 

Editor’s Note: This post was originally published in February of 2012 and has been updated for freshness, accuracy, and comprehensiveness.

 

The post How to Get a Perfect GMAT Score appeared first on Magoosh GMAT Blog.
This Blog post was imported into the forum automatically. We hope you found it helpful. Please use the Kudos button if you did, or please PM/DM me if you found it disruptive and I will take care of it. -BB
Magoosh GMAT Instructor
Joined: 28 Dec 2011
Posts: 4452
Own Kudos [?]: 28571 [1]
Given Kudos: 130
3 Common MBA Application Mistakes [#permalink]
1
Bookmarks
Expert Reply
FROM Magoosh Blog: 3 Common MBA Application Mistakes
In the past 9 years, we’ve worked on a lot of applications. Over 26,000 to be exact. And through that experience we’ve…

    1. made a lot of middle-of-the-night pots of coffee and

    2. seen some big trends across applications.
Surprisingly, year in and year out, countless applicants make the same pretty big mistakes as they approach their MBA applications. Mistakes that can really hold them back in the process. I polled the team of Admissionado MBAs to find out which are most prevalent so we can help you avoid them, and save time in the already overwhelming MBA application process.

Here are the 3 biggest offenders:

#1: Not answering the prompt
You’d be surprised by how prevalent this issue is. After all, responding to a very clear prompt seems like the easiest and clearest part of the MBA application process. But after polling our team of 40+ MBA experts, this was the number one response. And this issue comes in many forms:

  • Trying to get into the heads of the admissions committee members and just writing what you think they want to hear.
  • Wanting so badly to tell that one story that makes you look awesome, you just try to make it work in whatever essay you can, despite the question.
  • Trying to take essays for one school and smush them into essays for another.
  • Spending an entire career goals essay talking about your previous accomplishments and then rushing in a sentence about your short- and long-term goals at the very end.
Do not fall into this trap. Besides flat out lying in your applications, this is the fastest route to Ding City. Not only are the adcoms asking very specific questions for very specific reasons (read: they are looking to learn this specific thing about you so give it to them!), but you don’t want to come across as someone who can’t follow easy directions.

The adcom isn’t looking for a certain “type” of applicant, so don’t waste your time trying to be someone that you’re not. And if they want to know about your greatest accomplishment, they’re going to ask for it! So save yourself some time and give them what they are asking for. Literally. Answer the question they are asking.

#2: Underselling yourself in your resume
Let me save you from yourself right now and tell you that a job-hunting resume is not an MBA application resume. When you submit a resume in your apps, it should not merely be a list of all the things you’ve done. It should be a list of the things you’ve done and how you accomplished them and the impact of those accomplishments.

And don’t forget to put it all in context. Don’t make the mistake of assuming the person reading your application comes from your industry or even understands it. You need to present everything so anyone (even your grandma!) will know whether it is a big deal or not.

Your resume is your chance to highlight your leadership experience and your achievements, especially now that most MBA essays focus on the more personal side of things. You have very limited space to prove your impact, so you need to be clear, concise, and contextual.

#3: Not researching the individual programs.
If you’re about to drop $150K and 1-2 years of your life on something, it’s probably a good idea to do your due diligence. But even if you could see yourself being happy at any school with a strong brand, you still need to do your research. Why? To get in.

(Don’t believe me? Then listen to someone who is currently studying at MIT!)

When you’re applying to b-school, you need to prove to the adcom that you need this MBA in order to reach your career goals. And if you want to make that case, you need to also explain why that school – specifically – is going to get you there. Saying you like Columbia because it is in NYC is not enough. Neither is saying you like Kellogg because of their strong marketing program. You must go deeper. Get specific. Write a love letter to the school, laying out all the many ways it is going to get you exactly what you need to graduate, get a job, be super successful, and donate lots of money back to the school.

But you can’t do that without knowing the program inside and out.

Applying without doing research in is a waste of time. Yours and the adcom’s who will be reading that application. So take your time and do your research: talk to current students, talk to alum, visit the school… get as much information as you can.

The post 3 Common MBA Application Mistakes appeared first on Magoosh GMAT Blog.
This Blog post was imported into the forum automatically. We hope you found it helpful. Please use the Kudos button if you did, or please PM/DM me if you found it disruptive and I will take care of it. -BB
Magoosh GMAT Instructor
Joined: 28 Dec 2011
Posts: 4452
Own Kudos [?]: 28571 [0]
Given Kudos: 130
Princeton Review GMAT Book Review: Cracking the GMAT 2017 [#permalink]
Expert Reply
FROM Magoosh Blog: Princeton Review GMAT Book Review: Cracking the GMAT 2017
The newest Princeton Review GMAT book comes in two versions: Cracking the GMAT, 2017 Edition and Cracking the GMAT Premium, 2017 Edition. Both Princeton GMAT books are sparse on fundamentals and heavy on process of elimination, i.e. nifty test gimmicks. The premium version comes with six computer-adaptive practice tests, and costs about $7 more than the non-premium version on The Princeton Review’s website. (Though, of course, there are less expensive options on Amazon.)



Book images by The Princeton Review

The Princeton Review GMAT Book
When one reads these books, one may think that they are actually “cracking” the test: the questions are very straightforward, and The Princeton Review almost seems to be nudging you smugly saying, “See how easy this is.”

But the GMAT is not easy, and cannot be reduced to simple estimation and process of elimination. While both are useful techniques, to an extent, The Princeton Review attempts to predicate its entire pedagogy on what it fondly dubs POE: Process of Elimination.

Buyer Beware: The GMAT is far more difficult and requires learning the actual fundamentals and concepts behind the test.

I’m going to delve into the specifics of how the Princeton Review GMAT book handles Verbal and Math concepts and strategies. But first, let me preface both the Verbal and Math write-ups with a note: this organizational set up (sub-categories within Verbal and Math) is particularly suited to books that take the time to delve into each section independently. The Princeton Review skims the surface in each section, sometimes wasting an entire page with a single problem, or some distracting blurb in the margins telling us how the B-school student likes to wake up at 6:45 and watch CNN.

What is clearly missing from every section of the Princeton GMAT books is the ease-of-use and organization found in the Kaplan books. (That is not to say Kaplan is great; it too has numerous issues). Nonetheless, I’ve maintained the structure to highlight the different sections of the test and how The Princeton Review handles them.

Verbal
Critical Reasoning
The only section on the Verbal that The Princeton Review does to semi-decently is the Critical Reasoning section. The different question types are introduced. One learns how to take apart an argument. And one learns how to spot wrong answer choices. One doesn’t, however, get much practice. We learn the eight question types, but we do not even get eight practice questions. It’s funny how much the book promises on the cover, but how little it actually delivers.

Sentence Correction
Doing well on Sentence Correction requires learning both the fundamentals and advanced aspects of English grammar. To achieve this aim, the Manhattan GMAT has a 300-page book. The Princeton Review GMAT book tries to dispense Sentence Correction wisdom in a mere twenty pages (many of which do a poor job maximizing space).

To be fair, I could see this section being helpful to an absolute beginner. But even that person would need to learn far more grammar than that provided here, and would, at the very least, want more practice after learning about a specific grammar concept. (If there is a section on Parallelism it would be nice to have some practice questions). Ultimately, this points to the fact that this book is teaser. For the actual Princeton Review class? Maybe. My guess is they are hoping people purchase their other GMAT guides, those loaded with more questions.

Reading Comprehension

Again, The Princeton Review makes everything seem much easier than the test. Passages seem as though they were lifted from the encyclopedia. True, that level of reading can be warranted when a book is trying to impart a specific approach. Even then Princeton GMAT techniques are too general: at times I feel as though I am reading an SAT guide.

With The Princeton Review, unlike Kaplan, I do not feel l have learned much I can directly apply. Sure, I get the main idea of the passage and know to avoid extreme language in the answer choices…but really there is so much more to the GMAT than that. Terms such as ‘yin-yang’ tend to be more distracting than helpful.

That is not to say nothing is helpful here. But for an introduction to the GMAT, Kaplan does a far better job.

Math
For an absolute beginner, meaning somebody whose mind goes numb as soon as somebody so much as says the word ‘fraction’, Princeton GMAT offers an un-intimidating, straightforward approach to basic math. You get math drills, i.e. practice questions to help you learn the basics. Techniques such as “Picking Numbers” are introduced.

Unless you are looking to score about 500, the math help in this book will not help you attain a competitive score. Nevertheless, many of us are rusty in math, and in that sense, The Princeton Review provides a better primer than Kaplan (which already presupposes a decent level of math knowledge).

Problem Solving
The usual suspects: you’ll learn everything from mean, median, and mode to combinations. Again, everything is at a basic level to help reacquaint you in case you haven’t seen many of these concepts in awhile.

Data Sufficiency
Sure there is an entire section on Data Sufficiency, and even a section on advanced Data Sufficiency. The latter is not advanced, but targeted more at the medium-difficulty level. Again, same advice: if you are just beginning this will be helpful. That said, Kaplan does a far better job of introducing this tricky section. It essentially provides a framework to build off. The Princeton Review provides practice problems but not much of a framework.

Practice Tests
Much like the questions throughout the book, the test questions are a dumbed-down version of what you’ll see on the GMAT. That is not to say that these questions are poorly written. In fact, if you have a copy of this book, especially if you are just starting off, doing these practice questions wouldn’t hurt. The explanations are clear enough (though sometimes Princeton Review can be a little vague why they are eliminating a given answer – besides saying use process of elimination).

Analytical Writing Assessment (AWA)
Cracking the GMAT provides a decent introduction. Indeed if you are only looking to score about a 4 the templates the Princeton GMAT book provides may help you do so. I would have liked to see some example essays. Nonetheless, you can’t really fault any publisher for giving short shrift to the AWA, as most students are concerned only with their score out of 800.

Cracking the GMAT Pros
  • Provides a decent math refresher for those who haven’t seen math in a while.
  • Makes the test un-intimidating (a pro for those who are already stressed out enough at the prospect of taking the GMAT).
Cracking the GMAT Cons
  • Focuses far more on techniques than on fundamentals
  • Questions are far too easy compared to those found on the GMAT
  • Seems somewhat disorganized and in many sections does not provide a coherent framework
  • Nothing here at all for anyone looking to break 600
  • Large book creates an illusion of content, an appearance belied by the relative dearth of material
Princeton GMAT Grade: D+
 

Editor’s Note: This post was originally published in June of 2012 and has been updated for freshness, accuracy, and comprehensiveness.

The post Princeton Review GMAT Book Review: Cracking the GMAT 2017 appeared first on Magoosh GMAT Blog.
This Blog post was imported into the forum automatically. We hope you found it helpful. Please use the Kudos button if you did, or please PM/DM me if you found it disruptive and I will take care of it. -BB
Magoosh GMAT Instructor
Joined: 28 Dec 2011
Posts: 4452
Own Kudos [?]: 28571 [0]
Given Kudos: 130
GMAT Tuesday: Must Know Idioms #18 [#permalink]
Expert Reply
FROM Magoosh Blog: GMAT Tuesday: Must Know Idioms #18
Put on your vocabulary hats! It’s time to learn more idioms. In this video we’ll look at the idiomatic phrases involving responsible, ranging, agreeing, and defined.



Transcript
Hello! Welcome to GMAT Tuesdays. You know how everyone says you can hold a shell up to your ear and hear the ocean? Well. If you have an empty five gallon jug, you can do the same thing. Hello, welcome to another addition of Must Know Idioms. This is number 18!

Holy smokes, we’ve been doing a lot of these. So many idioms in the English language to know. We’ve been covering idioms because they pop up in the sentence correction section of the GMAT. And you’ll get tested on some very specific idioms, so we’re trying to cover those in these videos.

Idiom #1 – Responsible in vs. Responsible for
So let’s dive right in. The first idiom we’re going to look at is responsible in or responsible for. So this is an idiomatic expression, and the correct usage is responsible for.

So if you were to have a sentence like parents are responsible for their children’s education, that would be correct. You wouldn’t want to say parents are responsible in their children’s education. That is completely wrong.

Idiom #2 – Defined that vs. Defined as
Next, defined. If you see the word defined, are you going to use defined that or defined as? Well, you’re not going to want to use that. You’re always going to say defined as.

So for example, the president defined his role as leader of the country. That’s pretty straightforward. You wouldn’t want to say the president defined that his role was the leader of the country, that’s wordy and not idiomatically correct, so defined as.

Idiom #3 – Agreeing to vs. Agreeing with
Next, this is a tricky one, agreeing to or agreeing with. Which one is correct? Hm, it’s a toughy, because they both can be correct. It’s going to depend on the sentence.

So agree to or agreeing to means that you are accepting, or the person is accepting a duty, or accepting some responsibility. For example, Evan is agreeing to lead a meet up every other Wednesday at a local restaurant. I’ve accepted the responsibility to lead a group.

If we’re using with, this is about agreeing with someone’s opinion. So, say that someone says something and you agree with them. You think their idea is good, and you hold the same opinion. So, for example, like Marcy said that it was a bad idea to spend all of that money on a trip. And I would say I am agreeing with Marcy, I don’t think we should spend that money on a trip, I think we should spend it on… or put it away in our savings.

So when you see the word agreeing it’s really gonna depend on the context of the sentence. Agreeing to means that that person is taking on responsibility, the duty. Agree with means sharing same opinion.

Idiom #4 – Ranges from vs. Ranges of
Okay, that was a toughie. All right, finally, we have the word range. And do we wanna say ranges dot, dot, dot from or ranges dot, dot, dot of? We want to use from. Get rid of “of” here.

So for example, you could say Jeremy’s repertoire on the guitar ranges from classical to jazz standards, for an example. So you’re gonna use ranges from dot dot dot to.

All right, those are the four idioms for this week. Hope you find those useful. If you have any questions or need any help, please, please leave comments down below. And if you have any ideas for potential GMAT Tuesday videos I’m always looking for something new to do so leave comments about that as well and then finally if you need more help head over to gmat.magoosh.com where there are even more people like me ready to help you to dominate the GMAT.

All right, be excellent to the universe



The post GMAT Tuesday: Must Know Idioms #18 appeared first on Magoosh GMAT Blog.
This Blog post was imported into the forum automatically. We hope you found it helpful. Please use the Kudos button if you did, or please PM/DM me if you found it disruptive and I will take care of it. -BB
Magoosh GMAT Instructor
Joined: 28 Dec 2011
Posts: 4452
Own Kudos [?]: 28571 [0]
Given Kudos: 130
GMAT Quant: Rates and Ratios [#permalink]
Expert Reply
FROM Magoosh Blog: GMAT Quant: Rates and Ratios
What are rates and ratios? How do rate and ratios problems differ? Hint: Not that much.



Original photo by Pierre Rougier

In fact, rates are just ratios in disguise. Here are a four GMAT practice problems exploring rates and ratios.  Remember: no calculator!

Rates and Ratios Practice Problems
1) Someone on a skateboard is traveling 12 miles per hour.  How many feet does she travel in 10 seconds?  (1 mile = 5280 feet)

(A) 60

(B) 88

(C) 120

(D) 176

(E) 264

 

2) At 12:00 noon, a machine, operating at a fixed rate, starts processing a large set of identical items.  At 1:45 p.m., the twenty-first item has just been processed, and 15 have not yet been processed.  At what time will all 36 items be processed?

(A) 2:25 pm

(B) 3:00 pm

(C) 3:27 pm

(D) 4:13 pm

(E) 5:15 pm

 

3) An importer wants to purchase N high quality cameras from Germany and sell them in Japan.  The cost in Germany of each camera is E euros.  He will sell them in Japan at Y yen per camera, which will bring in a profit, given that the exchange rate is C yen per euro.  Given the exchange rate of D US dollars per euro, and given that profit = (revenue) – (cost), which of the following represents his profit in dollars?

(A) N(YC – DE)

(B) ND(YC – E)

(C) ND((Y/C) – E)

(D) N((Y/C) – DE)

(E) ND(Y – E)/C

 

4) Machine A and machine B process the same work at different rates.  Machine C processes work as fast as Machines A & B combined.  Machine D processes work three times as fast as Machine C; Machine D’s work rate is also exactly four times Machine B’s rate.  Assume all four machines work at fixed unchanging rates.   If Machine A works alone on a job, it takes 5 hours and 40 minutes.  If all four machines work together on the same job simultaneously, how long will it take all of them to complete it?

(A) 8 minutes

(B) 17 minutes

(C) 35 minutes

(D) 1 hour and 15 minutes

(E) 1 hours and 35 minutes

 

Solutions will come to these at the end of the article. Can’t contain your excitement? Click here to skip to the explanations.

Ratios and Proportions
Ratios are fractions.  When we have an equation of the form fraction = fraction, that’s called a proportion.   By far, the hardest part of dealing with a proportion is what you CAN and what you CAN’T cancel in a proportion.  Many students are quite confused on this issue.

First of all, let’s be clear that cancelling is simply division.  If I start with the fraction 24/32, and I “cancel the 8’s” to get 3/4, what I have really done is divide both the numerator and the denominator by 8.  Similarly, if I have 5/35, and I cancel the 5’s, in the numerator, I am left with 1: the simplified version is 1/7.  Too many student have the naïve view that canceling means “going away” or some other fairy-godmother operation.  Instead, cancelling is a card-carrying legitimate mathematical operation, the operation of division.

Clearly, it’s always legitimate to cancel in the numerator and denominator of the same fraction, the same ratio, so of course we can do that on each side in a proportion.



We might call that “vertical canceling” in a proportion: that’s 100% legal.  The one that often surprises folks is what we might call that “horizontal canceling” in a proportion, which looks like this:



Canceling a common factor from a & c would simply involve dividing both sides of the equation by the same number, a 100% legal move.   Similarly, canceling a common factor from b & d would simply involve multiplying both sides of the equation by the same number, another totally legal move.  Even though “horizontal canceling” across the equal sign may look suspect, it’s totally valid.

Now, the one that causes real problems is what we might call “diagonal canceling,” because so many students seem to be under the impression that is this OK, but in fact, it’s 100% illegal and incorrect.



I suspect people confuse this with “cross-canceling” in the process of multiplying fractions.  I actually abhor that uses term, “cross-canceling”: I think this term causes dozens of times more harm than good.  If we were to perform the canceling of a with d, that would essential be equivalent to dividing one side of an equation by a number and multiply the other side of the equation by the same number!  That’s not allowed!  We always have to do the same thing to both sides!  This is why this kind of “diagonal canceling” in a proportion is always disastrously incorrect.

OK, that’s the relevant mathematics without the real world stuff involved!

Rates
Rates are ratios, that is, fractions.  Any fraction with different units in the numerator and in the denominator is a rate: miles per hour, $ per pound, grams per cubic centimeter, etc.  Most rate questions can be solved by setting up a proportion.  One common proportion type involves a (part)/(whole) on each side: for example, part of the job over all of the job, and part of the price or time over all of the price or time. In setting up any rate proportion, we have to make sure that units match: the same units in the two numerators, and the same units in the two denominators.  The GMAT will expect you to know a few common unit changes (e.g. 1 hour = 60 min; 1 dozen items = 12 items, etc.); because some test-takers are familiar with metric and other are familiar with English, the GMAT most often would specify the conversion, as in #1 above.  In any case, a GMAT rate problem often involves reconciling units differences in some way before we can do the math.

Another pertinent topic is that of work rates.  Suppose Machine P does a job in 3 hours and Machine Q can do the same job in 6 hours.  How fast would it take both machines working together?  You see, we can’t add or subtract the times it takes to perform jobs.  What we can add are the work rates!  It doesn’t matter that these work rates would have the ambiguous units of “job/hour”—it doesn’t matter as long as every rate in the problem has the same units.  The rate of P, job per time, would be 1/3, which means either one job every three hours or one-third of a job every hour: either is correct.   Similarly, the rate of Q would be 1/6.  We can’t add or subtract times, but we can add individual work rate to find a combined work rate.  Adding fractions, we get (1/6) + (1/3) = (1/6) + (2/6) = 3/6 = 1/2.   The combined rate of P & Q working together is 1/2, or one job per 2 hours.  Thus, if P & Q were working together, it would take them just two hours to get the job done.  That is the basic logic of work rates.

Rate and Ratios Summary
If you understand the rules of fractions and the concept of work rate, there’s nothing about rates and ratios you can’t understand.  If you had any “aha” moments while reading this article, give the practice problems above another look before jumping in the solutions below.



proportions, we can cancel a common factor in the two numerators; cancel the factor of 7.



Now, cross-multiply, and use the doubling & halving shortcut for multiplying.

T = 15*12 = 30*6 = 180

Now, 180 minutes = 3 hours, so the task finishes 3 hours later, at 3 p.m.

Answer = (B)

 

3) All the other currencies are related to euros, so we should focus on getting everything to euros and then changing it all at once to dollars.

Remember that profit = revenue – cost.  For one camera, cost is E euros.  The revenue of one camera is Y yen: let’s change that to euros, so that we can express cost, revenue, and profit all in euros.

We have an exchange rate of C yen/euro, with yen in the numerator and euros in the denominator.  If we were to multiply this, we could cancel euros and wind up with yen.  We don’t want that.  We want to cancel yen and wind up with euros, so we need to divide by C.  Y/C is the revenue of one camera in euros.

This means that ((Y/C) – E) is the profit in euros of one camera.

Now, the other exchange rate is D dollars/euro, with dollars in the numerator and euros in the denominator.  If we multiply this, we cancel euros and get dollars.  That’s exactly what we want.  Thus, D((Y/C) – E) is the profit, in dollars, of one camera.

Now, just multiply by the number of cameras: ND((Y/C) – E)

Answer = (C)

 

4) Let A, B, C, and D be the rates of Machines A, B, C, and D respectively.  We know that

(i) C = A + B

(ii) D = 3C

(iii) D = 4B

Starting with (ii) and (iii), equate the two expressions equal to D, and then substitute in the expression from (i) equal to C.

4B = 3C = 3(A + B) = 3A + 3B

B = 3A

Then, C = A + 3A = 4A, and D = 3*(4A) = 12A

The combined rate,

A + B + C + D = A + 3A + 4A + 12A = 20A

Since the combined rate is 20 times faster than Machine A alone, the combined time should be divided by 20.

Machine A alone takes 5 hr 40 min, or 340 minutes for the whole job.  Divide this by 20:

340/20 = 17

The combination of the four machines will take 17 minutes to complete the job.

Answer = (B)

 

 

Editor’s Note: This post was originally published in August, 2014 and has been updated for freshness, accuracy, and comprehensiveness.

The post GMAT Quant: Rates and Ratios appeared first on Magoosh GMAT Blog.
This Blog post was imported into the forum automatically. We hope you found it helpful. Please use the Kudos button if you did, or please PM/DM me if you found it disruptive and I will take care of it. -BB
Magoosh GMAT Instructor
Joined: 28 Dec 2011
Posts: 4452
Own Kudos [?]: 28571 [0]
Given Kudos: 130
The Official Guide for the GMAT 2017 Book Review [#permalink]
Expert Reply
FROM Magoosh Blog: The Official Guide for the GMAT 2017 Book Review
As you may be aware, GMAC, the folks who create the GMAT, recently released three volumes of The Official Guide for the GMAT 2017. I will review these new editions of the Official Guide for the GMAT in this book review.

The Official Guide for the GMAT 2017
The three new volumes are as follows:



1) The Official Guide for the GMAT 2017 (white cover)

2) The Official Guide for the GMAT Verbal Review 2017 (pink cover)

3) The Official Guide for the GMAT Quantitative Review 2017 (blue cover)

As it turns out, #1 was littered with mistakes.  We have a blog about the corrections to this guide.  GMAC found all the mistakes, and put out a new version that (theoretically) has everything corrected.



4) The Official Guide for the GMAT 2017, Corrected (green cover)

FACT: Each one of #1-3 of these replaces a corresponding 2016 version published about a year ago.

FACT: Each one of #1-3 has about 25% new content, compared to its 2016 correlate.

FACT: Version #4 has no new content, just the same content as in #1 but with no typos!

Another new Official Guide?
As readers of this blog may know, I have the highest respect for the GMAT exam as one of the finest standardized tests in existence. As a consequence, I have the highest respect for the content creators and psychometricians at GMAC who design this test. I have met some of these people, and they are quite impressive individuals.

Having said that, GMAC is a company, and like any company, it has a tendency to leverage what it has to generate profits. In the “old days” (up until a couple years ago), they would publish a new OG every 3-4 years, and often they would have a particularly good reason to do so.  For example, they published the OG13 when they were releasing the then-new Integrated Reasoning section in 2012: that was a 100% legitimate reason to update the OG.  In the past couple years, they have started publishing a new OG every year, and they are rushing each new edition out so fast that the last one (version #1 above) was full of mistakes.  Thus, if you bought the #1, they would be happy to sell you #4 as well, even though the content is identical.  This new-OG-every-year rhythm is clearly being driven much more by pure profit chasing, rather than by any legitimate pedagogical concerns.  It’s basically a ploy to separate the vulnerably anxious test-taking population out there from as much of their money as possible.   Caveat emptor.

I will point out that the newest OG 2017, like the OG 2016 and OG 2015, have all the questions in the book also online, if you want to practice them on a computer rather than from the print version.  Furthermore, that online question bank is where they keep the practice Integrated Reasoning questions.

Should I buy the new Official Guide?
Putting the criticisms aside, I will address the most pertinent question to an individual test taker reading this blog: should I, the student studying for the GMAT, buy these new books?

Here’s what I’ll say. If you are just starting your studies for the GMAT, if you haven’t bought any official materials yet, then yes, by all means, you should buy some version of the GMAT OG, and you might as well buy the newest one available.

If you already have an earlier edition, the OG 2016 or even the OG13 or OG2015, and are already working through it, then I would say that definitely is good enough. If you master everything in either one of those volumes, that’s still enough for a high 700s score. After all, the GMAT itself hasn’t changed since the introduction of IR in 2012. The new OG may be marginally more GMAT-like, but I am NOT going to say that it’s so revolutionarily better that you should throw away the previous edition and run out to buy the new one. Undoubtedly, some marketers at GMAC would love it if a large number of students thought that way, but with all due respect, I want to discourage this line of thinking.

If you have already finished working through the OG2016, and need more practice questions, that would be another reason to buy the new guide, because about 25% of the questions are new, not repeats from the previous edition. Similarly, if you exhausted an earlier edition studying for a first take of the GMAT, and now you need to study for a retake, then the new questions in one of the earlier editions would help you.

What about the Verbal Review and Quant Review?
These are similar enhancements over the earlier editions. If you only have about a month to study for the GMAT, you probably wouldn’t have time to do any questions other than the OG questions. Even in some of our three month study schedules, folks barely have enough time simply to learn and review just the content they need to master: they don’t have time for these extra questions.

If you are a practice-question maven who has already raced through the OG and you need more official questions, or if you exhausted the OG on your first take and now you want to practice for a retake, then these books are an excellent source of more official practice questions.

If you already have the earlier editions, by all means, use those first. Only buy these new books if you don’t already own the earlier editions.

OG 2017 Book Review: Summary
Understandably, most students studying for the GMAT want to do everything in their power to prepare. By all means, use the best resources, follow proven study schedules, and pursue the habits of excellence (without which the resources & study schedules are considerably less valuable!) All that is very important. Nevertheless, don’t feel compelled to leap for your credit card every single time GMAC publishes a new edition of something. The OG 2017 is a collection of absolutely excellent GMAT practice questions, but the same description also applies to the two previous editions. If you are starting from scratch, you might as well start with the newest. If you already have an earlier GMAT OG, trust the one you have.

If you have any experience with using any of these new books, we would love to hear from you in the comments section below.

 

Editor’s Note: This post was originally published in March, 2012 and has been updated for freshness, accuracy, and comprehensiveness.

nbsp;

The post The Official Guide for the GMAT 2017 Book Review appeared first on Magoosh GMAT Blog.
This Blog post was imported into the forum automatically. We hope you found it helpful. Please use the Kudos button if you did, or please PM/DM me if you found it disruptive and I will take care of it. -BB
Magoosh GMAT Instructor
Joined: 28 Dec 2011
Posts: 4452
Own Kudos [?]: 28571 [0]
Given Kudos: 130
The Official Guide for the GMAT 2017 Book Review [#permalink]
Expert Reply
FROM Magoosh Blog: The Official Guide for the GMAT 2017 Book Review
As you may be aware, GMAC, the folks who create the GMAT, recently released three volumes of The Official Guide for the GMAT 2017. I will review these new editions of the Official Guide for the GMAT in this book review.

The Official Guide for the GMAT 2017
The three new volumes are as follows:



1) The Official Guide for the GMAT 2017 (white cover)

2) The Official Guide for the GMAT Verbal Review 2017 (pink cover)

3) The Official Guide for the GMAT Quantitative Review 2017 (blue cover)

As it turns out, #1 was littered with mistakes.  We have a blog about the corrections to this guide.  GMAC found all the mistakes, and put out a new version that (theoretically) has everything corrected.



4) The Official Guide for the GMAT 2017, Corrected (green cover)

FACT: Each one of #1-3 of these replaces a corresponding 2016 version published about a year ago.

FACT: Each one of #1-3 has about 25% new content, compared to its 2016 correlate.

FACT: Version #4 has no new content, just the same content as in #1 but with no typos!

Another new Official Guide?
As readers of this blog may know, I have the highest respect for the GMAT exam as one of the finest standardized tests in existence. As a consequence, I have the highest respect for the content creators and psychometricians at GMAC who design this test. I have met some of these people, and they are quite impressive individuals.

Having said that, GMAC is a company, and like any company, it has a tendency to leverage what it has to generate profits. In the “old days” (up until a couple years ago), they would publish a new OG every 3-4 years, and often they would have a particularly good reason to do so.  For example, they published the OG13 when they were releasing the then-new Integrated Reasoning section in 2012: that was a 100% legitimate reason to update the OG.  In the past couple years, they have started publishing a new OG every year, and they are rushing each new edition out so fast that the last one (version #1 above) was full of mistakes.  Thus, if you bought the #1, they would be happy to sell you #4 as well, even though the content is identical.  This new-OG-every-year rhythm is clearly being driven much more by pure profit chasing, rather than by any legitimate pedagogical concerns.  It’s basically a ploy to separate the vulnerably anxious test-taking population out there from as much of their money as possible.   Caveat emptor.

I will point out that the newest OG 2017, like the OG 2016 and OG 2015, have all the questions in the book also online, if you want to practice them on a computer rather than from the print version.  Furthermore, that online question bank is where they keep the practice Integrated Reasoning questions.

Should I buy the new Official Guide?
Putting the criticisms aside, I will address the most pertinent question to an individual test taker reading this blog: should I, the student studying for the GMAT, buy these new books?

Here’s what I’ll say. If you are just starting your studies for the GMAT, if you haven’t bought any official materials yet, then yes, by all means, you should buy some version of the GMAT OG, and you might as well buy the newest one available.

If you already have an earlier edition, the OG 2016 or even the OG13 or OG2015, and are already working through it, then I would say that definitely is good enough. If you master everything in either one of those volumes, that’s still enough for a high 700s score. After all, the GMAT itself hasn’t changed since the introduction of IR in 2012. The new OG may be marginally more GMAT-like, but I am NOT going to say that it’s so revolutionarily better that you should throw away the previous edition and run out to buy the new one. Undoubtedly, some marketers at GMAC would love it if a large number of students thought that way, but with all due respect, I want to discourage this line of thinking.

If you have already finished working through the OG2016, and need more practice questions, that would be another reason to buy the new guide, because about 25% of the questions are new, not repeats from the previous edition. Similarly, if you exhausted an earlier edition studying for a first take of the GMAT, and now you need to study for a retake, then the new questions in one of the earlier editions would help you.

What about the Verbal Review and Quant Review?
These are similar enhancements over the earlier editions. If you only have about a month to study for the GMAT, you probably wouldn’t have time to do any questions other than the OG questions. Even in some of our three month study schedules, folks barely have enough time simply to learn and review just the content they need to master: they don’t have time for these extra questions.

If you are a practice-question maven who has already raced through the OG and you need more official questions, or if you exhausted the OG on your first take and now you want to practice for a retake, then these books are an excellent source of more official practice questions.

If you already have the earlier editions, by all means, use those first. Only buy these new books if you don’t already own the earlier editions.

OG 2017 Book Review: Summary
Understandably, most students studying for the GMAT want to do everything in their power to prepare. By all means, use the best resources, follow proven study schedules, and pursue the habits of excellence (without which the resources & study schedules are considerably less valuable!) All that is very important. Nevertheless, don’t feel compelled to leap for your credit card every single time GMAC publishes a new edition of something. The OG 2017 is a collection of absolutely excellent GMAT practice questions, but the same description also applies to the two previous editions. If you are starting from scratch, you might as well start with the newest. If you already have an earlier GMAT OG, trust the one you have.

If you have any experience with using any of these new books, we would love to hear from you in the comments section below.

 

Editor’s Note: This post was originally published in March, 2012 and has been updated for freshness, accuracy, and comprehensiveness.

nbsp;

The post The Official Guide for the GMAT 2017 Book Review appeared first on Magoosh GMAT Blog.
This Blog post was imported into the forum automatically. We hope you found it helpful. Please use the Kudos button if you did, or please PM/DM me if you found it disruptive and I will take care of it. -BB
Magoosh GMAT Instructor
Joined: 28 Dec 2011
Posts: 4452
Own Kudos [?]: 28571 [0]
Given Kudos: 130
Save money and time with these GMAT resources… [#permalink]
Expert Reply
FROM Magoosh Blog: Save money and time with these GMAT resources…


The amount of time GMAT students spend researching the best prep options is rivaled only by the amount of money spent on unnecessary (and occasionally sub-par) GMAT prep materials. This stops now.

Magoosh’s mission is to make test prep accessible to all. One way that we achieve this is by creating, finding, and aggregating top GMAT prep materials for our students.

When researching and developing GMAT prep resources, we keep several facts in mind:

1. Successful GMAT prep requires excellent resources

2. Sometimes the best GMAT prep books aren’t the newest editions

3. GMAT students are busy, therefore prep needs to be efficient

4. GMAT students are on a budget, therefore prep needs to be affordable

Given this information, we compiled a comprehensive list of the best GMAT books and resources available today. Some are free and some require payment. All are worth the investment of time and money.

Click here to view our newest resource.

Your prep starts and stops with this material.

Happy Studying!

 

The post Save money and time with these GMAT resources… appeared first on Magoosh GMAT Blog.
This Blog post was imported into the forum automatically. We hope you found it helpful. Please use the Kudos button if you did, or please PM/DM me if you found it disruptive and I will take care of it. -BB
Magoosh GMAT Instructor
Joined: 28 Dec 2011
Posts: 4452
Own Kudos [?]: 28571 [0]
Given Kudos: 130
The Official Guide for the GMAT 2017 Book Review [#permalink]
Expert Reply
FROM Magoosh Blog: The Official Guide for the GMAT 2017 Book Review
As you may be aware, GMAC, the folks who create the GMAT, recently released three volumes of The Official Guide for the GMAT 2017. I will review these new editions of the Official Guide for the GMAT in this book review.

The Official Guide for the GMAT 2017
The three new volumes are as follows:



1) The Official Guide for the GMAT 2017 (white cover)

2) The Official Guide for the GMAT Verbal Review 2017 (pink cover)

3) The Official Guide for the GMAT Quantitative Review 2017 (blue cover)

As it turns out, #1 was littered with mistakes.  We have a blog about the corrections to this guide.  GMAC found all the mistakes, and put out a new version that (theoretically) has everything corrected.



4) The Official Guide for the GMAT 2017, Corrected (green cover)

FACT: Each one of #1-3 of these replaces a corresponding 2016 version published about a year ago.

FACT: Each one of #1-3 has about 25% new content, compared to its 2016 correlate.

FACT: Version #4 has no new content, just the same content as in #1 but with no typos!

Another new Official Guide?
As readers of this blog may know, I have the highest respect for the GMAT exam as one of the finest standardized tests in existence. As a consequence, I have the highest respect for the content creators and psychometricians at GMAC who design this test. I have met some of these people, and they are quite impressive individuals.

Having said that, GMAC is a company, and like any company, it has a tendency to leverage what it has to generate profits. In the “old days” (up until a couple years ago), they would publish a new OG every 3-4 years, and often they would have a particularly good reason to do so.  For example, they published the OG13 when they were releasing the then-new Integrated Reasoning section in 2012: that was a 100% legitimate reason to update the OG.  In the past couple years, they have started publishing a new OG every year, and they are rushing each new edition out so fast that the last one (version #1 above) was full of mistakes.  Thus, if you bought the #1, they would be happy to sell you #4 as well, even though the content is identical.  This new-OG-every-year rhythm is clearly being driven much more by pure profit chasing, rather than by any legitimate pedagogical concerns.  It’s basically a ploy to separate the vulnerably anxious test-taking population out there from as much of their money as possible.   Caveat emptor.

I will point out that the newest OG 2017, like the OG 2016 and OG 2015, have all the questions in the book also online, if you want to practice them on a computer rather than from the print version.  Furthermore, that online question bank is where they keep the practice Integrated Reasoning questions.

Should I buy the new Official Guide?
Putting the criticisms aside, I will address the most pertinent question to an individual test taker reading this blog: should I, the student studying for the GMAT, buy these new books?

Here’s what I’ll say. If you are just starting your studies for the GMAT, if you haven’t bought any official materials yet, then yes, by all means, you should buy some version of the GMAT OG, and you might as well buy the newest one available.

If you already have an earlier edition, the OG 2016 or even the OG13 or OG2015, and are already working through it, then I would say that definitely is good enough. If you master everything in either one of those volumes, that’s still enough for a high 700s score. After all, the GMAT itself hasn’t changed since the introduction of IR in 2012. The new OG may be marginally more GMAT-like, but I am NOT going to say that it’s so revolutionarily better that you should throw away the previous edition and run out to buy the new one. Undoubtedly, some marketers at GMAC would love it if a large number of students thought that way, but with all due respect, I want to discourage this line of thinking.

If you have already finished working through the OG2016, and need more practice questions, that would be another reason to buy the new guide, because about 25% of the questions are new, not repeats from the previous edition. Similarly, if you exhausted an earlier edition studying for a first take of the GMAT, and now you need to study for a retake, then the new questions in one of the earlier editions would help you.

What about the Verbal Review and Quant Review?
These are similar enhancements over the earlier editions. If you only have about a month to study for the GMAT, you probably wouldn’t have time to do any questions other than the OG questions. Even in some of our three month study schedules, folks barely have enough time simply to learn and review just the content they need to master: they don’t have time for these extra questions.

If you are a practice-question maven who has already raced through the OG and you need more official questions, or if you exhausted the OG on your first take and now you want to practice for a retake, then these books are an excellent source of more official practice questions.

If you already have the earlier editions, by all means, use those first. Only buy these new books if you don’t already own the earlier editions.

OG 2017 Book Review: Summary
Understandably, most students studying for the GMAT want to do everything in their power to prepare. By all means, use the best resources, follow proven study schedules, and pursue the habits of excellence (without which the resources & study schedules are considerably less valuable!) All that is very important. Nevertheless, don’t feel compelled to leap for your credit card every single time GMAC publishes a new edition of something. The OG 2017 is a collection of absolutely excellent GMAT practice questions, but the same description also applies to the two previous editions. If you are starting from scratch, you might as well start with the newest. If you already have an earlier GMAT OG, trust the one you have.

If you have any experience with using any of these new books, we would love to hear from you in the comments section below.

 

Editor’s Note: This post was originally published in March, 2012 and has been updated for freshness, accuracy, and comprehensiveness.

 

The post The Official Guide for the GMAT 2017 Book Review appeared first on Magoosh GMAT Blog.
This Blog post was imported into the forum automatically. We hope you found it helpful. Please use the Kudos button if you did, or please PM/DM me if you found it disruptive and I will take care of it. -BB
Magoosh GMAT Instructor
Joined: 28 Dec 2011
Posts: 4452
Own Kudos [?]: 28571 [0]
Given Kudos: 130
Kaplan GMAT Premier 2017 Book Review [#permalink]
Expert Reply
FROM Magoosh Blog: Kaplan GMAT Premier 2017 Book Review
If you’re wondering whether you should by the newest version of the Kaplan GMAT prep book, then you’ve come to the right place. Kaplan GMAT prep has come out with the GMAT Premier 2017 with 6 Practice Tests book, the newest in their arsenal of GMAT review books. I’ll offer my honest review so that you can decide whether to spend your money (about $30 on Amazon) here or look for another GMAT prep resource.



Kaplan GMAT Premier 2017: Overview
Each year, the major test prep companies trot out the same test prep guides. Exact same content, typos and all — but with one major change: the glossy cover will now have the most recent year on it and a new stock photo model with a forced smile. Unless the test undergoes a major overhaul, which hasn’t happened since the IR section (more of an add-on, by the way), then we can go 15 years with the exact same content, albeit with strikingly different hairstyles on the person smiling away deliriously on the cover.

Yet Kaplan has really worked in the last few years at improving their GMAT guide. They’ve added a diagnostic test at the beginning and this year they even added 40 advanced GMAT Quant questions. For that, I commend them for bucking the trend of simply slapping a new cover on an old book.

And this new guide might have been the perfect “one stop shop” for your GMAT needs were it not for the questions. Indeed, at a superficial level, that is exactly what I imagine a perfect GMAT prep guide to look like: you start off with a diagnostic test (much as the Official Guide does), you offer easy-to-follow explanations (as Kaplan does here) and you break up each section with clearly laid out strategies and practice questions in which you can apply these very strategies.

But success on the GMAT is very much about doing well on GMAT questions–and the questions in this book aren’t that GMAT-like.

Kaplan GMAT: Verbal
Grade: D

This lack of GMAT question verisimilitude especially goes for the Verbal questions, which, to be frank, are not worth your time. Sure, you might get some mileage out of the practice strategies mentioned in this book, but the questions are too easy.

Let’s take Sentence Correction questions. First off, the sentence structure is too basic. You aren’t going to get those multiple clauses chained together and sophisticated punctuation such as em-dash, semicolons, and colons are all but absent. And that semi-sophisticated register of a GMAT question (meaning it sounds like it was written by a college professor) has been replaced by a voice that it sounds more like a well-read high school senior.

On the more difficult Sentence Correction questions, you’ll get two long, long answer choices that are exactly the same, save for one word. Essentially, you’ll have to hunt for that one word. The GMAT will never do this. And so this question, much like most of the difficult Kaplan questions, are difficult in an un-GMAT like way.

So what? Well, the skills you use on the hard official questions aren’t getting sharpened. Instead of looking for such nuances as awkward phrasing or faulty idiom, you brain begins to approach the GMAT in “scanning mode” — A big no-no.

This is but one example. I could go on and on, Kaplan GMAT verbal question after verbal question, pointing out how none of the answer choices are staggered in the way in the GMAT would and that many lack the sophistication of the trap answers. And how about most of these questions end up being flat out easy — something you just won’t see on the GMAT. Or, I can tell you the save your time and money, and stick to the verbal questions in the Official Guide.

There is the Critical Reasoning section, in which I could spend another 1000 words describing why it is subpar and why going through it would likely hurt — or at least not help — your score. (You can ask me in the comments if you want to know more). But basically the content is far too easy. Again, use the Official Guide. Much the same can be said for the reading passages.

Does that mean all of the Kaplan verbal is bunk? No. For the absolute beginner, the lessons are instructive and do provide a reasonable foundation — albeit a basic one. But even that student would be better off totally ignoring the practice questions in Kaplan and picking up a copy of the Official Guide.

Kaplan GMAT Prep: Math
Grade: C

The good news is that, in terms of practice content, the Kaplan Math is slightly better. But that’s mostly because the Verbal is so subpar. I’d say most of the questions in both the diagnostic test and the mini-lessons are good practice for somebody starting out. You’ll get to practice concepts that might be rusty for you and try practice questions that to help you reinforce those concepts. What newbies won’t get from the Kaplan GMAT Premier 2017 with 6 Practice Tests book is how to crack GMAT math questions — that is, the specific methods that can save you lots of time (how to eliminate wrong answers, backsolving and plugging in, estimation, etc.)

Then, there are those folks who are not beginners, who have have already spent time with other GMAT sources, mainly official material. If you are in that group, I have one thing to say to you: stay away from this book.

See, you’re probably in the market for the hard stuff done in the GMAT style. But it’s when Kaplan tries to do difficult questions that we end up more with questions that are in the tedious brain teaser category and are totally inelegant. They are not at all the kind of questions GMAT would actually ask. So, essentially, you aren’t getting any questions that are actual hard GMAT questions. It’s only on these questions that you can really practice. By doing the GMAT Kaplan questions, you’ll develop an approach that works for these questions. You’ll look for “Kaplan-like tricks” on the official questions. What you won’t be looking for are the real GMAT tricks. This will hurt both the GMAT beginner and one who has worked for months with official GMAT questions.

Kaplan GMAT Book Review Takeaway
I’m not saying that you should only use official material, that only the GMAC can get it right. For example, if you compare this to the Quant in the Manhattan GMAT Quant prep, you’ll see that, while even harder than the actual GMAT, the writers of the Manhattan questions are seasoned adepts at the GMAT and know the test very well.

The person writing this Kaplan book doesn’t really get all the nuances of the test. So the tough questions, especially those in the advanced GMAT Quant section, are likely to be more frustrating than enlightening. Again, for tough questions, done in the GMAT vein, head over to Manhattan GMAT (this company also provides an Advanced Quant guide section that while super hard is still in line with the GMAT style).

And for any who claims that I’m some grumpy test prep dude out to get Kaplan, keep in mind that Kaplan owns Manhattan GMAT, so a sale for the latter is a sale for the former. I just want to make sure students out there are using the best material, and staying away from the subpar stuff.

Final Word on the Kaplan GMAT Book
Overall Grade: C-

 

Editor’s Note: This post was originally published in June, 2012 and has been updated for freshness, accuracy, and comprehensiveness.

 

The post Kaplan GMAT Premier 2017 Book Review appeared first on Magoosh GMAT Blog.
This Blog post was imported into the forum automatically. We hope you found it helpful. Please use the Kudos button if you did, or please PM/DM me if you found it disruptive and I will take care of it. -BB
Magoosh GMAT Instructor
Joined: 28 Dec 2011
Posts: 4452
Own Kudos [?]: 28571 [0]
Given Kudos: 130
3 Tips for Deciding Which Business Schools are a Good Fit for You [#permalink]
Expert Reply
FROM Magoosh Blog: 3 Tips for Deciding Which Business Schools are a Good Fit for You
With so many MBA programs to choose from, how do you pick the right one for you? In other words, which b-schools are the best “fit” for you? “Fit” is established by how well your needs and desires correspond to what a school has to offer. What elements are you looking for in an MBA program? Does School X offer these elements? You also need to look at what applicant qualities School X is looking for. Do you have these traits?

Applying to schools without the proper fit will be a waste your time, effort, and money. Since, if you are not a good fit, you will most likely not be accepted. And if you are accepted and decide to go, you probably won’t get as much out of the program as you could.

Here are three tips to help you find the b-schools that fit you best:

    1. Assess your career goals and educational needs. Analyze your work experience and think about what you want your future to look like. Examine the classes, tracks, and research centers at your programs of interest and decide if they match up to your interests and goals. Speak with current students and alumni about their experiences, goals, and work prospects/careers. Do the school’s strengths match up with your goals?

    2. Evaluate your qualifications as an MBA applicant. Visit the websites of programs that seem interesting to you and look at their class profiles. Make sure that your GMAT score, GPA, and years on the job are within the parameters set by the school. If they aren’t, you will be less competitive at that school, and will probably be a poor fit.

    3. Study the intangibles. Make use of the school websites and student profiles. Can you envision yourself working, studying and learning with these students? If yes, this school could be a great fit for you. Not all schools have the same philosophy and approach to their b-school curriculum. Think about how you learn and determine if their courses will fit your needs. If you can, visit the programs and sit in on classes. If you feel comfortable with the teaching method and philosophy, that’s another point in favor of your fit with the school. Other features – including class size, how approachable the professors are, and the environment of the school (its size, location, etc.) – will also help determine your fit with a specific b-school.
Once you’ve decided what elements are important to you and weighed them all, you can decide how a school will fit you. Keep in mind that fit is relative. Not all of the features that you want in a school are equally weighted, and you will likely be more willing to compromise on some than others. Choosing the programs that are most in line with your personality and career goals can help you avoid a costly mistake – attending a program that does not meet your needs. And careful research up-front will also help you identify programs where your stats and experience make you a competitive applicant – a good fit from the school’s perspective. The goal is to find a program that’s a great fit for you, and where you’re a great fit.

For more tips on applying to the b-schools with your best fit, download our free guide, Best MBA Programs: A Guide to Selecting the Right One.

The post 3 Tips for Deciding Which Business Schools are a Good Fit for You appeared first on Magoosh GMAT Blog.
This Blog post was imported into the forum automatically. We hope you found it helpful. Please use the Kudos button if you did, or please PM/DM me if you found it disruptive and I will take care of it. -BB
Magoosh GMAT Instructor
Joined: 28 Dec 2011
Posts: 4452
Own Kudos [?]: 28571 [0]
Given Kudos: 130
GMAT Tuesday: OG Reading Comprehension #3 [#permalink]
Expert Reply
FROM Magoosh Blog: GMAT Tuesday: OG Reading Comprehension #3
Question 3 in the reading comprehension section of the Official Guide to the GMAT is again, a detail question. They’ve loaded up the detail questions in this first group of questions.
We’ll dive into how to approach detail questions, how to identify wrong answers, and of course, how to arrive at the right answer.



Transcript
Well hello. You caught me in the middle of me caring for my beautiful succulent garden. Just give it a little bit of water every seven to ten days. Okay, let’s get to GMAT Tuesdays. Welcome, thanks for joining me for another GMAT Tuesdays. We are diving into the official guide to the GMAT and working through reading comprehension questions and we’re on question number three.

Passage Review: GMAT Official Guide Reading Comprehension Question #3
In a previous video we went through the passage and attacked the passage. As a refresher of what the passage is all about I wrote down the main idea structure and tone here. So the main idea is the passage about 2 ideas or 2 theories of schooling. The structure, first paragraph is the introduction.

Second paragraph is about the theory A or Idea A. Paragraph number 3 is about the critics of theory A and then introduces theory B which talks about the confusion effect. And the fourth paragraph is all about theory B. And then the tone, it’s balanced-ish. And then basically the author is sort of unbiased here, generally just presenting the two theories, not arguing for one or the other.

Official Guide Reading Comp Question #3
Okay so like I said we’re in question number 3 which is a detail question. And you might ask Kevin, how do you know it’s a detail question? That is an excellent question. So in the passage, we’re in this book. There’s newer ones but you’ll find this question in the new ones as well.

According to the explanation, so any time I see that phrase according to the passage according to the explanation, I know I’m dealing with a detail question. I’ll continue reading. According to one explanation of the confusion effect, a fish that swims in a school will have a greater advantage for survival if it does something.

So now that I know I’m dealing with a detail question, I know what my process should be, or my approach to this question. So, we’ve already attacked the passage. We can rephrase the question. They’re asking about the confusion effect and they want to know what sort of advantage the confusion effect gives a school of fish or what theory B says the advantage is.

I would do some research in the passage so I know from the structure that I wrote down that I’ll probably be focused on the fourth paragraph. That’s where there’s two sort of benefits that are described of the confusion effect. Then I would anticipate the answer, and then finally, eliminate any wrong answers.

Remember, we want to work on eliminating wrong answers first instead of just trying to find the right answer. I find it easier to eliminate wrong answers cause there’s more of them. And it helps narrow in on what is right about the right answer. And it gets you thinking about how something is wrong.

Sometimes there’s only gonna be one small thing in an answer choice that’s wrong, and you need to find that one small thing in order to eliminate it. And that helps you to really confirm the right answer. If you can’t find anything wrong it, then you know you found the right answer.

So, there are common wrong answers in detail questions. One, is an inference where the answer choice synthesizes information from two different places in the passage. Another one is distortion where it takes information from the passages and twists it. And then finally anytime you see an answer choice that just adds new information even if it’s on the same topic, that’s gonna be wrong. There has to be something in the passage that says what the answer choice is saying. If you don’t find that, then it’s going to be wrong answer.

Answer Choices
Okay, let’s take a look at our answer choices. So answer choice A, says that, so remember we’re talking about theory B, and it’s talking about being visible at 200 meters.

This is a perfect example of a distortion answer choice. Because this description is of theory A, not theory B. So they want to make sure that you are searching the right part of the passage and not just finding information that’s in the passage, you have to know what the question is asking. So, I know I’m looking for a theory B information, that was theory A.

Answer choice B talks about the fish being in the front of the group or in the rear of the group. There’s nothing in the passage that talks about the placement of the fish in the schooling group, so I know this is new information and wrong.

Answer choice C talks about whether it’s a small group or a large group. Again, the passage doesn’t go into the size of the schooling group. If you look at the fourth paragraph, there’s nothing about how big the school is in terms of it helping to protect them from predators so this is new information again and wrong.

In answer choice D, it talks about how the fish are all similar. They’re all similar in appearance which is discussed in line 34. So this is definitely in the passage, so this is our right answer.

Finally, answer choice E, they talk, really short answer, it just says medium-sized basically. Again, there’s no information about the size of the fish or the size of the group discussed in the passage so this is new information and we can consider it wrong.

All right, that is it. If you have any questions, please feel free to leave comments down below. And if you need more help with the GMAT head over to magoosh.com and we’d be happy to help.

All right, be excellent to the universe.



The post GMAT Tuesday: OG Reading Comprehension #3 appeared first on Magoosh GMAT Blog.
This Blog post was imported into the forum automatically. We hope you found it helpful. Please use the Kudos button if you did, or please PM/DM me if you found it disruptive and I will take care of it. -BB
Magoosh GMAT Instructor
Joined: 28 Dec 2011
Posts: 4452
Own Kudos [?]: 28571 [0]
Given Kudos: 130
GMAT Sentence Correction: the “Due To” Mistake [#permalink]
Expert Reply
FROM Magoosh Blog: GMAT Sentence Correction: the “Due To” Mistake
This mistake is astonishingly common in spoken English, even among otherwise relatively well-spoken people.  In the six Sentence Correction practice questions below, all the questions contain “due to“: where is it used correctly and where it is wrong?

 

1) Elysium Field Construction planned to build a ten-story building the suburban downtown, but due to unstable bedrock in that region, the entire project may have to be canceled.

(A) due to unstable bedrock in that region, the entire project may have to be canceled

(B) the cancellation of the entire project may be brought about, due to unstable bedrock in that region

(C) cancelling the entire project, an unavoidable consequence, because of unstable bedrock in that region

(D) with the unstable bedrock in that region, the entire project is possibly canceled

(E) unstable bedrock in that region may necessitate the cancellation of this entire project.

 

 

2) None of the typical physical properties, such as color, hardness, and melting point, of the element Astatine (At) are known because, due to its high radioactivity, it almost immediate decays into something else.

(A) known because, due to its high radioactivity, it almost immediately decays into something else

(B) known due to the fact that it decays almost immediately into something else because of its high radioactivity

(C) known if, because of its high radioactivity, it almost immediate decays into something else

(D) known: because of its high radioactivity, it almost immediate decays into something else

(E) known: due to its high radioactivity, it almost immediately decays into something else

 

 

3) The motion of the Sun in the sky, as well as the Moon and planets and stars, are because of the rotation of the Earth: everything rises in the East and sets in the West.

(A) are because of

(B) is because of

(C) are caused by

(D) is due to

(E) are due to

 

 

4) According to classical Marxism, the class struggle, caused by economic inequity, bringing about the universal workers’ revolution that would end history.

(A) struggle, caused by economic inequity, bringing about the universal workers’ revolution that would end history

(B) struggle, due to economic inequity, would bring about the universal workers’ revolution, thereby ending history

(C) struggle, which was brought about by economic inequity, causing the universal workers’ revolution that would end history

(D) struggle was because of economic inequity, and it would bring about the universal workers’ revolution and the end of history

(E) struggle was due to economic inequity, bringing about the universal workers’ revolution and ending history

 

 

5) In 1865, the then-revolutionary sound of Wagner’s opera Tristan und Isolde was due to the relative atonality of the so-called “Tristan chord.”

(A) In 1865, the then-revolutionary sound of Wagner’s opera Tristan und Isolde was due to the relative atonality of the so-called “Tristan chord.”

(B) In 1865, Wagner created what was then a revolutionary sound in his opera Tristan und Isolde, this sound caused by the relative atonality of the so-called “Tristan chord.”

(C) In 1865, the relative atonality of the so-called “Tristan chord” was causing the the then-revolutionary sound of Wagner’s opera Tristan und Isolde.

(D) Because of the relative atonality of the so-called “Tristan chord,” Wagner’s 1865 opera, Tristan und Isolde, had a then-revolutionary sound.

(E) The relative atonality of the so-called “Tristan chord” has caused the then-revolutionary sound of Wagner’s 1865 opera Tristan und Isolde.

 

 

6) In the Battle of Roncevaux Pass in 778, the rearguard of the Frankish army, under the command of Roland, was obliterated due to an ambush by the Basque army, an event made famous in La Chanson de Roland.

(A) In the Battle of Roncevaux Pass in 778, the rearguard of the Frankish army, under the command of Roland, was obliterated due to an ambush by the Basque army, an event made famous in La Chanson de Roland

(B) In the Battle of Roncevaux Pass in 778, the Basque army ambushed the Frankish army, the rearguard of the Frankish army, under the command of Roland, was obliterated, and the event was made famous in La Chanson de Roland

(C) La Chanson de Roland made famous the obliteration, due to an ambush by the Basque army, of the rearguard of the Frankish army, under the command of Roland, at the Battle of Roncevaux Pass in 778

(D) In La Chanson de Roland, the Basque army ambushed the rearguard of the Frankish army, under the command of Roland, and obliterated them, and this event was famous at the Battle of Roncevaux Pass in 778

(E) In the Battle of Roncevaux Pass in 778, the rearguard of the Frankish army, under the command of Roland, ambushed and obliterated by the Basque army, an event made famous in La Chanson de Roland

 

Complete explanations will follow this article.

 

The word “due”
The word “due” is an adjective.  Admittedly, it is used rarely in its unadorned form as a noun-modifier, although you may be familiar with the idea of someone performing some task “with due diligence” or a health care professional giving someone “due care and attention.”

This adjective idiomatically takes the preposition “to” in a idiom of causality.  To say that “P is due to Q” is to say that Q played some role in causing P.

 

A mistake due to “due to”
The construction “due to” has a meaning quite similar to the meaning of “because of.”  The mistake is to treat the former as if it were identical to the latter.  It is not.

The construction “due to” is an adjective that idiomatically takes a prepositional phrase: as an adjective, the word “due” is a noun-modifier and most modify a target noun, according to the rules of noun modifiers.  In particularly, it follows the Modifier Touch Rule patterns, typically touching the target noun it  modifies.

By contrast, the construction “because of” is a compound preposition that would open a prepositional phrase.  This preposition phrase would be a verb modifier.  Remember that verb modifier target the action of a clause and are much freer in their placement than are noun modifiers: in particular, there is no “touch rule” or anything like this for verb modifiers.  Thus, “because of” correctly can be used just about anywhere in the clause and it can directly modify the action of the clause.

The “due to”  mistake is using “due to” as if it were a verb modifier, as if it were able to be placed freely and modify the action of a clause.  As a noun modifier, it only can modify nouns.  The construction “due to” can be placed correctly next to a target noun, (“The crisis, due to the prime minister’s death, was …”) or it can be used correctly after a form of the verb “to be”—this latter construction is called a predicate adjective, a term you don’t need to know (“The noise is due to traffic.”) It’s a mistake, though, and a very common mistake at that, to use “due to” to modify the action of a clause.

Thus



In the above exemplary sentences, only sentence #7 exemplifies the “due to” mistake, because the noun modifier “due to” is attempting to modify the verb.  Sentence #8 rewrites #7 with the correct verb modifier modifying the verb.  Sentence #9 is a correct construction, using “due to” as a predicate adjective: P is due to Q.  Sentence #10 is funny: the verb modifier is trying to modify a noun and, thus, the entire sentence sounds awkward.  In #11, the noun modifier “due to rain” modifies the subject, “cancellation.”  In #12, the meaning changes: the verb modifier “because of rain” is modifying the verb, “cost”: this make it sound as if the it’s not the cancellation itself that costs money but only the fact that it occurred in the rain.  In other words, #12 is 100% grammatically correct, but it means something quite different from #11.

The construction “due to” and “because of” are similar in meaning, but it is a mistake to use them as if they were interchangeable.  Such mistakes are sprinkled throughout the practice questions above.

 

Summary
If the article above gave you any insights, you may want to look at the questions above again before jumping into the explanations below.  Here’s another practice question:

13) Senator Charles Sumner

Also, check out our idiom flashcards.  I hope your understanding of this topic has increased—because of this blog!!



Practice Problem Explanations
1) Choice (A) commits the classic “due to” mistake: the action of being cancelled cannot be modified by a noun modifier.  Choice (A) is incorrect.

In (B), again the noun modifier “due to” is trying to modify the verb “brought about.” Choice (B) is also indirect and awkward, so it is incorrect.

Choice (C) commits the famous missing verb mistake: we have an independent clause, then the word “but,” and we are expecting another full independent clause, but after the word “but” there is not full verb.  Choice (C) is incorrect.

Choice (D) is ambiguous: who or what is “with the unstable bedrock“? The role of this prepositional phrase is not clear.  Also, “is possibly canceled” is an awkward way to convey this idea.  Choice (D) is incorrect.

Choice (E) is not ideal, not the most elegant possible phrasing, but it is grammatically and logically correct.  It presents an independent clause that correct states the causal relationship of the ideas.

Choice (E) is the best answer of these five.

 

 

2) A question about the elusive 85th element, Astatine.

Choice (A) commits the classic “due to” mistake: the action of decaying cannot be modified by a noun modifier.  Choice (A) is incorrect.

Choice (B) also commits the classic “due to” mistake: here, the noun modifier cannot modify the action situation of not being known.  Choice (B) is incorrect.

Choice (C) changes the meaning: it is as if the sentence is suggesting that the element has a choice about whether to decay immediately.  This is not the original meaning.  Choice (C) is incorrect.

Choice (D) correctly uses the verb modifier “because of” to modify the verb “decays.”  This is a promising choice.

Choice (E) commits the third “due to” mistake in the problem: once again, the action of decaying cannot be modified by a noun modifier.  Choice (E) is incorrect.

 

 

3) Split #1: SVA.  Everything following the words “as well as” is what is called an “additive phrase”: these do not count as part of the subject.  The subject is the singular word “motion,” so the verb must be singular.  Choices (A), (C), and (E) are incorrect.

Split #2: this is the predicate adjective structure: [noun]”is”[adjective].  It is a mistake to put a verb modifier in the place of this adjective.  Choices (A) & (B) make this mistake.  Choices (D) & (E) correct have a noun modifier after the verb, so these are correct in this regard.  Choice (C) uses a slightly different structure, a passive verb “are cause by,” which could be correct if it didn’t have an agreement problem.

The only possible answer is (D).

 

 

4) Choice (A) commits the famous missing verb mistake: the bonafide subject “class struggle” never gets a bonafide verb, so this is not a complete sentence on its own.  Choice (A) is incorrect.

Choice (B) correctly uses “due to” to modify a noun, and this version is grammatically correct and logically clear.  This is a promising choice.

Choice (C) also commits the famous missing verb mistake: we get a complete modifying clause, but the main clause has no verb.  Choice (C) is incorrect.

Choice (D) makes the mistake of using “because of” after a form of the verb “to be.”  The verb modifier cannot be used as a predicate adjective.  Choice (D) is incorrect.

Choice (E) is grammatically correct but a bit awkward.  The “due to” structure in the predicate adjective is 100% correct.  The logical relationship between the “universal worker’s revolution” and the “end of history” is unclear: they are presented side-by-side, as two different things, rather than as logically linked.  Choice (E) is incorrect.

The only possible answer is (B).

Disclaimer: Magoosh, as a private company that earns a profit in a free-market economy, clearly does not endorse Marxism.  As a bunch of peaceful Berkeley folks, we certainly don’t want to see any violent revolutions toward or against anyone!  Instead, to mitigate a few of the inequities of international capitalism, we offer stratospherically high quality test prep at very affordable prices: that’s our revolution!!  

 

5) A question about Wagner‘s great opera, Tristan und Isolde, and its famous opening chord.  BTW, this is one of Mike’s favorite operas!

Choice (A) is completely correct.  This correctly uses the predicate adjective structure for “due to”: P is due to Q.

Choice (B) is grammatically correct but a bit indirect and awkward.  It is a much wordier way to convey the same idea.  If all the other answers were wrong, this might be acceptable, but this is not as elegant as (A).  Choice (B) is incorrect.

Choice (C) is grammatically correct but the choice of the progressive verb is quite unusual: was the chord causing the revolutionary sound only in 1865? The logical implications of the progressive verb are unclear, but this one seems to depart from the intended meaning.  Choice (C) is incorrect.

Choice (D) is completely correct.  This correctly uses the verb modifier “because of” to modify the action of a verb.

In choice (E), the use of the present perfect tense, “has caused,” is awkward: it doesn’t logically fit with the rest of the sentence.  It would make sense of the effect were something today, not something over a century ago.  Choice (E) is incorrect.

We are down to a choice between (A) and (D).  Of these two, (A) is rhetorically more effective: it is elegant and direct: the structure “is due to” is a very clear structure.  The main verb of (D) is simple “had”—that’s the heart of the action? Boring!  Overall, choice (D) is not bad, but choice (A) is better, and is the best answer here.

 

 

6) A question about the Battle of Roncevaux Pass, in which the Basque ambushed Charlemagne‘s army.  According to legend, Roland, who died in the ambush, was Charlemagne’s nephew.  The Song of Roland, about the battle, is the oldest surviving major work in the French literary.

Choice (A) commits the classic “due to” mistake: the action of being obliterated cannot be modified by a noun modifier.  Choice (A) is incorrect.

Choice (B) commits the mistake of false parallelism: it mechanically puts all the information into parallel, with absolutely no regard for logic.  Choice (B) is incorrect.

Choice (C), while not the most elegant, is grammatically correct.  The noun modifier “due to” correctly modifies a noun.

Choice (D) is grammatically correct but illogical.  It speaks about the action as if it only occurred in the poem, and then was famous at the battle(?).  In the original sentence, the action was the battle!  This logically scrambles the relationships of the ideas.  Choice (D) is incorrect.

Choice (E) commits the famous missing verb mistake: the “rearguard of the Frankish army” is a bonafide subject, but this is never followed by a bonafide verb, just participles.  This is not a complete sentence.  Choice (E) is incorrect.

Choice (C) is the only possible answer.

 

The post GMAT Sentence Correction: the “Due To” Mistake appeared first on Magoosh GMAT Blog.
This Blog post was imported into the forum automatically. We hope you found it helpful. Please use the Kudos button if you did, or please PM/DM me if you found it disruptive and I will take care of it. -BB
Magoosh GMAT Instructor
Joined: 28 Dec 2011
Posts: 4452
Own Kudos [?]: 28571 [0]
Given Kudos: 130
GMAT Tuesday: OG Reading Comprehension #4 [#permalink]
Expert Reply
FROM Magoosh Blog: GMAT Tuesday: OG Reading Comprehension #4
This is the final video in our series on the reading comprehension section of the Official Guide to the GMAT. In this final question, we are confronted with a main idea question. I talk about how to approach these quesitons, how to identify the common wrong answers, and finally, how to find the right answer.

 

Transcript
Hello! Welcome to GMAT Tuesdays. I was just doing a little finger painting. I don’t know if you know, but I’m an expert finger painter. Look at this beautiful ocean-scape that I just did. Isn’t that nice? There’s a whale jumping out of the ocean. All right, enough painting, enough art, down to business.

Let’s talk about the official guide to the GMAT and more reading comprehension questions. We are looking at question number four, which is the last question in this series. We’ve done one, two, three, and looked at the passage.

Passage Review: GMAT Official Guide Reading Comprehension Question #4
So as a reminder of the passage, the main idea here is schooling. There’s two theories. The structure. The first paragraph is the introduction, second paragraph is all about the theory A. Third paragraph is critics of theory A, introduction of theory B. And the fourth paragraph is about theory B. And then finally, the tone is balanced-ish. And the author is basically just presenting two ideas. I wouldn’t say the author is biased or has a real stake in any one of them.

Official Guide Reading Comp Question #4
Okay, so question number 4 is a main idea question, and that’s because the question says, what is the primary purpose of the passage? That is a very clear clue for a main idea question.

That means that our approach is going to be attack the passage, which we did. Rephrase the question. You never really need to rephrase these, but basically just know what’s the main idea of this passage. What’s great is you could easily skip number 3 if you’re taking good notes and have a good sense of the passage.

I already know that it’s about two theories of schooling we’ve been talking about that. And then, the real key is just going through the answer choices and eliminating anything that is wrong. So, in order to find wrong answers, it’s good to know the common wrong answers for main idea questions.

So, one common wrong answer is the distortion. So, adding a word in there that makes the answer wrong when everything else there is right, that would be a distortion. Extreme words or language. This is, you’ll notice these if you’re paying attention to the tone of the passage and the author’s purpose.

The extreme answer choices will stand out because there’s something there that just doesn’t sound right with the tone of the passage. And then anytime that there is a detail from the passage so it’s something that’s in the passage but doesn’t really encompass everything that the passage is about. That’s going to be wrong. And then finally, anything new if it’s on the topic but not really about what’s in the passage then that’s gonna be wrong as well.

Answer Choices
Okay, so let’s look at our answer choices. Answer choice A, says that the passage is presenting two different theories. Wow, great. That’s exactly what we’re looking for. And we’ve been talking about how the passage presents two ideas. There isn’t really any bias there and it’s just describing these two theories. So answer choice A is our answer.

Now let’s look at these wrong answers. Answer choice B talks about analyzing techniques. This is new information, there’s nothing about techniques in the passage. There is some analysis but no, you’re not analyzing techniques, you’re looking at theories.

Finally, C.) defending ideas, this is a distortion. There are ideas but the author again isn’t bias, isn’t arguing from one idea over the other so there’s no defense. That is a distortion.

Answer choice D, refuting beliefs. This is extreme and a little bit of a distortion too. You might thing beliefs could also be theories, but they’re not they’re different. And then refuting, again, refuting is extreme. There’s no refutation in the passage at all, it’s just a presentation.

Finally, enter choice E revealing new evidence. This would be cool evidence, I said ividence, that’s weird. So this is new information, there’s nothing in the passage about some new reveal that we haven’t heard from or knew about before. The theories that are discussed seemed to have been talked about for awhile. One of them has been around long enough for there to be critics of it, so this is wrong as well.

All right, we’re all done. That was it, so easy. All right, if you need any help with what we’ve talked about, feel free to leave comments down below.

And also feel free to head over to gmat.magoosh.com for even more help with your GMAT studies. All right, be excellent to the universe.



The post GMAT Tuesday: OG Reading Comprehension #4 appeared first on Magoosh GMAT Blog.
This Blog post was imported into the forum automatically. We hope you found it helpful. Please use the Kudos button if you did, or please PM/DM me if you found it disruptive and I will take care of it. -BB
Magoosh GMAT Instructor
Joined: 28 Dec 2011
Posts: 4452
Own Kudos [?]: 28571 [0]
Given Kudos: 130
5 Reasons to Synchronize GMAT Prep with B-School Applications [#permalink]
Expert Reply
FROM Magoosh Blog: 5 Reasons to Synchronize GMAT Prep with B-School Applications
It’s a common approach that most MBA applicants follow. Start off with test prep, spend several months on it, take the test, and then start thinking about what to do next.

It’s true that this strategy (of taking up each component in the application requirements and tackling them one at a time) allows you to focus entirely on the task at hand and reduce the distractions. However, there are a few drawbacks that you should be aware of.

Keep in mind that you are putting together multiple pieces of a bigger jigsaw puzzle.

  • There are many things in the process that have an interdependency on other components. We’ll consider some of them in the sections below.
  • A linear process (finishing the GMAT first before considering the other elements) stretches the application timeline significantly.
  • Preparing for the GMAT/ GRE is an intense process. Your fatigued brain needs a break to recharge and refocus. Taking up other crucial aspects of the application process can help you do just that.
Here’s what you can do in conjunction with (and not after) your GMAT prep, as you continue to aim for that super high GMAT score.

1. Having a business school list ready can save you money.

Even if you aren’t applying immediately after you get a GMAT score, you do have the privilege of sending your scores to 5 programs for free. Any other business school that you include in your application list will incur an additional cost.

Of course, you could walk into the test center knowing all the tricks to crack quantitative problems, yet nothing about your prospective business schools. There are no penalties for that. However, having a short list of 5 schools will surely give you a little bonus.

2. Deciding if you need an MBA consultant can help you plan your timeline.

Just like you decided whether you’ll sign up (or not) for professional help for your test preparation, there’s another question waiting to be answered. Should you hire an MBA consultant?

Before you make that decision, it would help to dig a little deeper. Consultants can come in all hues. The scope of work they do varies and — unlike MBA rankings — there isn’t any globally accepted list of good consultants. Spend some time trying to understand what they can do for you, what they shouldn’t do for you (like writing your essay) and what you should do on your own.

Why is it important to know this upfront? Because working with a consultant will make the process slower (and not the other way around) and impact your timeline. That’s a strong reason to do the research along with your GMAT prep.

Here are some more thoughts on whether hiring MBA admission consultants is worth it or not.

3. Learning about the MBA essay topics early will give you time to introspect.

Just like you’ve prepared to take any quant or verbal problems, take some time to understand what business schools will throw at you in their essay topics. The essay prompts may often sound deceptively simple since they cover more or less the same concepts (why MBA, why now, career goals).

Since there are no right or wrong answers to these questions, it’s tough to know if you are on the right track. It can take weeks or months of introspection to figure out some of the answers. Don’t assume all the answers will fall in place when you are ready to start writing the essays.

4. Researching the various career options will strengthen your application.

Though you won’t need this for your test, you will need to have clarity about where you are heading with your MBA. What doors do you want the degree to open up for you? Do you even require an expensive MBA to get there? Will you enjoy your new career in that industry and role? Or will you end up looking for an exit a few years after enthusiastically getting into it?

Read up on the list of career options. Learn about the various industries. If possible, talk to MBA alumni who are working in those roles. Decide if you want a specialist or a generalist career. You’ll find a lot of interesting opportunities in the industry that offer a logical extension to what you’ve been doing. Don’t assume consulting and banking are the only options you have.

5. Learning about the education loan process can help you avoid heartburn later.

Most MBA students don’t have pockets deep enough to fund their degree without some external help from funding agencies. The student loan process advertised on various websites may seem straight-forward—drop your application at the entrance, go shopping at the mall, on the way out get your loan approval. But in reality, takes longer than that.

The process can also involve getting your head around several unusual terms (like APR, collateral, moratorium) that you may not be familiar with. Don’t rely on one financing team just because your friend got funded by them. Start talking to multiple companies. Find out whether you’d be eligible to apply and the documents you’d need.

In summary, when you are applying to the top programs there’s a whole lot of work to be done apart from religiously following your GMAT study plans. Don’t defer it all till the last moment. It’ll just mean that you’ll end up cutting corners to meet the application deadlines.

Start early. Take small steps. The bigger picture will seem less intimidating when the application submission dates draw closer.

The post 5 Reasons to Synchronize GMAT Prep with B-School Applications appeared first on Magoosh GMAT Blog.
This Blog post was imported into the forum automatically. We hope you found it helpful. Please use the Kudos button if you did, or please PM/DM me if you found it disruptive and I will take care of it. -BB
Magoosh GMAT Instructor
Joined: 28 Dec 2011
Posts: 4452
Own Kudos [?]: 28571 [0]
Given Kudos: 130
4 Tips for Deciding How Many Business Schools to Apply To [#permalink]
Expert Reply
FROM Magoosh Blog: 4 Tips for Deciding How Many Business Schools to Apply To
You’ve decided to apply to business school. You’ve done your research to see which schools support your goals and meet your needs, and have come up with a long list…too long a list. You know you can’t apply to all of them. This is not an easy decision, but we’re here to help you with four tips for choosing the right number of b-schools to apply to.

It Depends

Before we present our tips, you should know that the question of how many b-schools to apply to is best answered with the classic “It depends” answer. It depends on your abilities, how much time and money you’ve set aside for applications, and how well you can deal with the possibility of being rejected more than a few times. It also depends on who you ask. Some authorities stand by applying to 3-4 schools, without exception. Our guidelines will provide you with some more flexibility.

Breaking Down the Numbers

1. If you have 2-3 schools on your list that are a good match for your goals, have acceptance rates of more than 30%, and are schools where you’re strongly competitive, then applying to 2-3 schools makes complete sense. There’s really no need to apply to any more than that – you will likely get into at least one of these programs with a solid application.

2. If you discover that only 2-3 schools support your goals and meet your needs, and you decide that if you were not to get into these programs, you’d forgo your b-school plans for this year and reapply next year, then you should go ahead and apply to just these schools, even if they’re reaches for you. The idea here is that if you know that you won’t go to a school that’s easier to get into (i.e. a safety school), then there’s really no point in wasting your time and money applying. This doesn’t happen frequently, but it does happen.

3. If you have target schools that you dream of attending but that have acceptance rates under 15% and/or your credentials aren’t competitive, then you should apply to 1-2 of these and 3-6 where you’re more likely to be admitted. Of course, after reading #2, you know that you must be open to attending one of these safer schools, or there’s simply no point in applying. You must also make sure that these safety schools still support your goals.

4. If your target schools have acceptance rates of less than 30% and you’re competitive but not guaranteed admission (like most people), then it would make sense to apply to 3-6 schools in this range and 1-2 in the range where you’re very likely to be accepted.

Evaluating Your Goals & Credentials

Your goals and credentials relative to others competing for your spot in b-school are of the utmost importance. If you belong to an overrepresented group (like Indian IT males) you should consider applying to slightly more schools. You also don’t need to apply to all of your schools in the first round. You can divide your applications between first and second round deadlines.

A Final Word

There are those who say that applying to 6-8 schools is too expensive and time-consuming, and that it will detract from your ability to produce excellent applications. This would probably be true if you were to complete all these applications in one month. However, if you want to increase your chance of acceptance and can submit your applications over two rounds, then this number can be realistic. It’s an option for you to consider.

As I said before, it depends.

Still not sure how many schools you should apply to or where you should apply? Get the answers you need when you check out Accepted’s easy-to-use guide, Best MBA Programs: A Guide to Selecting the Right One. Grab your free copy here!

The post 4 Tips for Deciding How Many Business Schools to Apply To appeared first on Magoosh GMAT Blog.
This Blog post was imported into the forum automatically. We hope you found it helpful. Please use the Kudos button if you did, or please PM/DM me if you found it disruptive and I will take care of it. -BB
Magoosh GMAT Instructor
Joined: 28 Dec 2011
Posts: 4452
Own Kudos [?]: 28571 [0]
Given Kudos: 130
What Is a Good GMAT Score? [#permalink]
Expert Reply
FROM Magoosh Blog: What Is a Good GMAT Score?
What is a good GMAT score and who defines this abstruse and perplexing standard? Can good GMAT scores and average GMAT scores differ by a single point? Do business schools agree on a single definition of good and bad scores at some top-secret GMAT Scores for Business Schools Convention?



When it comes to discussing the question of what’s a good GMAT score versus a bad score, there are a lot of questions to answer (see above). But the most important thing to know is that a good GMAT score for one person is not necessarily the same as a good GMAT score for another person. Your goal GMAT score may be 10 points above or below that of your friend or peer. So, the question really becomes: What is a good GMAT score for you?

Before we get into a discussion of setting a target GMAT score for yourself, let’s take a step back and discuss GMAT scores in broad strokes.

GMAT Scores: Percentiles
The GMAT is scored from 200 to 800. Two-thirds of students score between 400 and 600 on the exam. According to GMAC, the folks who create the GMAT, the GMAT score percentiles reveal the link between GMAT score and percentile of everyone who takes the GMAT. To start, here are some of the correlations between GMAT score and percentile.

The mean GMAT score is 551.94.

ScorePercentile Ranking

80099%

75098%

70089%

65076%

60059%

55043%

50030%

45019%

40011%

3506%

3003%

2502%

2000%

Sample Size: 757,035

Standard Deviation: 120.88

Data Period: 2013 – 2015

Note: GMAC has not yet released percentile data for 2016

Notice that 780 and 790 and 800 all mean about the same in the great scheme of things.  What constitutes a “good GMAT score” to some extent depends on what you mean.

If you score anywhere over 600, you have done better than the majority of folks who take the GMAT—you have an above average GMAT score but far from a perfect score.  If you score over 600, and certainly if you score over 650, that will be high enough to get you into reasonably respectable schools.  What, though, if you have set your sights higher?

What is a good GMAT score for top business schools?
Each year, US News & World Report ranks the “Best Business Schools”, and if you sign up with them, you can get the full information for these schools (tuition, enrollment figures, average GMAT scores, average undergraduate GPA, acceptance rates, and percent of students employed at graduation).  Harvard and Stanford top the list.  The 2016 average GMAT scores for students at these two universities are 725 and 733 respectively.

Remember, those are average GMAT scores, which means that individual scores at each of those schools vary both above and below those numbers.  If your GMAT score is, say, 740, then it would be above-average for every business school in the world.  For the other “top ten” schools, the average GMAT scores are between 715 and 733.  If you score above 710, your score is in the territory of the elite schools, and if you score anywhere above 750, your GMAT score is stratospherically high.  At that point, business school admission depends far more on the other aspects of your application, especially your work experience, your references, your interview, and your essays.  A high, even perfect, GMAT score will not help you if you have no valuable work experience or cannot make a compelling case for yourself.

For more information on the GMAT scores needed for top business schools, I highly recommend taking a look at our GMAT Scores for Top Business Schools infographic.



Perfect GMAT score, average GMAT score, or in between
If you are currently at, say, 600, getting up to 650 would be a huge move—a push from the 59th percentile to the 76th percentile.  If you are at, say 680, then getting up to 710 would be enormous—crossing the great 700 threshold, moving from top 25% to top 10%.

BUT, if you already have scored between 710-750, adding another 30 points to your GMAT score really won’t do much for your application—and if all the extra blood & sweat & tears it takes to get that additional 30 points take away from the rest of your application, it’s not worth it.  With a GMAT in the 710-750 zone, you have already abundantly demonstrated your academic ability is quite sufficient to prosper at Wharton, Sloan, or Kellogg.  There are other dimensions you need to demonstrate as well.

If you take the GMAT once, and score higher than 750, that’s great.  If you take it once, get a 720, and want to take it again in an attempt to score higher, think again.  There’s a diminishing returns problem here.

In simple terms, once your GMAT score is more than about 700, the “academic achievement” box is checked.  The schools know you can handle the academic load—both a 720 and a 770 make that basic same statement.  What matters after that is whether the rest of your application is well-rounded—whether are you are good fit for the school and a person they can imagine with a promising leadership potential.  If you have that, then all you need from the GMAT, even for the top schools, is something in the 700+ range; if you don’t have the well-rounded stuff, adding another 50 points to an already high GMAT score will not do bupkis for your application.

Once you get a 700+ GMAT score, it’s insanity to spend more time trying to improve it: at that point, you are done with the GMAT, and your efforts should be directed to making the rest of your application demonstrate that you are an overall well-rounded candidate.  Getting a 770 GMAT score is a neat trick, but if that is the only thing you have to your credit, you are just a “one trick pony” as far as elite business schools are concerned.

Want a higher GMAT score?
What if the hazards of the stratosphere are not your concern? If you have little idea of your own starting point, I would suggest starting with the Magoosh GMAT Diagnostic Test.  

What if your GMAT score is currently in the low 600’s and you would like to move to the high 600s or even low 700s?  Read through the articles on this free blog and check out some of our resource recommendations.  We have a series of study schedules you may find helpful—check them out here. Also, check out our review of the best GMAT books and resources!  Your personal best GMAT score is not necessarily a perfect score, but it’s what you can do when you are fully prepared and fully on your game—that’s exactly what Magoosh can do for you!

So, what’s a good GMAT score for you?
Ultimately, you need to identify the GMAT score you would need to get into your target program or programs. Many students apply to target schools, safety schools, and reach schools. At the very least, you should apply to programs that you have a reasonable chance of getting into. Once you have your list of the schools you’d like to apply to, do some research:

  • Identify your target school
    • Research the programs you’re interested in attending and make a list of the business schools you plan to apply to.
  • Research the program’s application requirements
    • Go to the school’s admissions website and figure out what their application entails. Make note of important application deadlines so that you know how long you have to prepare for your GMAT and complete the rest of your application requirements.
  • Research stats on the program’s most recently admitted class
    • GMAT score ranges can vary widely from one program to another. Most business schools will openly state the average GMAT score of their recently admitted class on their admissions website. Some even offer GMAT score percentile ranges for the recently admitted class. Start there. You can always subscribe to US News & World Report to get more information if you can’t find it for free.
  • Talk to current students and the admissions committee
    • In addition to learning more about the program and the culture of the school, talking with current students and the admissions committee can help you gain insight into the admissions process. Maybe your chosen program values your essay and recommendation letters more than a top 20% GMAT score. Or maybe you really do need a 710 to get in. Either way, it’s useful to know how things work.
  • Set your target GMAT score
    • In order to increase your chances of admission, you’re going to want to aim for a GMAT score that is higher than the average score of the recently admitted class. If you’re scoring in the 75th percentile of students, then you’re in good shape. If you’re in the 50th percentile, then you’re going to either need to try for a higher score or spend a lot of time perfecting the rest of your application.
  • Prep with your target GMAT score in mind
    • Determine your baseline score, sign up for a prep course, start using a study schedule, take timed practice tests, and be sure to focus your energy on your weaknesses, not on re-affirming your strengths.
GMAT Scores: Summary
Who knew that determining your “good GMAT score” would be a challenge in and of itself? Hopefully you now have a better idea of how it’s accomplished and can go about researching your target schools and setting your goal GMAT Score.

So tell us—what are your GMAT score aspirations?  What are your plans for the GMAT and for business school?  What has been your experience in the B-school application process?  What about taking the GRE instead? Maybe check out this GMAT to GRE score conversion to see where you would stand. Anyway, we would love to hear from you in the comments below!

 

Editor’s Note: This post was originally published by Mike McGarry in February of 2013, and has been recently updated by Rita Kreig for freshness, accuracy, and comprehensiveness.

The post What Is a Good GMAT Score? appeared first on Magoosh GMAT Blog.
This Blog post was imported into the forum automatically. We hope you found it helpful. Please use the Kudos button if you did, or please PM/DM me if you found it disruptive and I will take care of it. -BB
GMAT Club Bot
What Is a Good GMAT Score? [#permalink]
   1   2   3   4   5   6   7   8   9   10   11  ...  14   

Powered by phpBB © phpBB Group | Emoji artwork provided by EmojiOne